日本高考题

一根木棒从墙角滑落时扫过的区域是??(保持两端分别在直角两边上)——大阪大学2011年高考第二题(理科)

首先考虑一个问题:一个长为 1 的木棒 AB 靠在墙角。 A:(0,1),B:(0,0) 。向右轻轻推一下木棒,木棒开始滑落。在滑落过程中, A,B 始终分别在 y 轴正半轴和 x 轴正半轴上。滑落过程中木棒扫过的区域是什么样的?

xy 平面上有两个动点 P:(0,sin\theta),Q:(8cos\theta,0) 。当 \theta 在 0\leq\theta\leq\frac{\pi}{2} 范围内变化,设变化过程中线段 PQ 扫过的范围为 D ,求 D 绕着 x 轴旋转一周得到的立体的体积 V 。

 

如果把整个坐标系在 x 轴方向缩小到原来的 \frac{1}{8} ,那么 D 也会缩小 \frac{1}{8} (缩小后的领域记作 E )。这时, 对于 P_0:(0,sin\theta),Q_0:(cos\theta,0) 来说, E 就是线段 P_0Q_0 在移动过程中所扫过的领域。

 

注意, P_0Q_0=\sqrt{cos^2\theta+sin^2\theta}=1 ,是定长的。

 

因此原题等同于

 

一个长为 1 的木棒 P_0Q_0 靠在墙角。 P_0:(0,1),Q_0:(0,0) 。向右轻轻推一下木棒,木棒开始滑落。在滑落过程中, P_0,Q_0 始终分别在 y 轴正半轴和 x 轴正半轴上。求滑落过程中木棒扫过的区域 E 在 x 轴方向扩大 8 倍得到的区域 D 以 x 轴为轴旋转一周所得到的立体的体积。

 

蛮有意思的一道题,这个领域会是什么呢?

————————————————————————————————————————

 

首先列出直线 P_0Q_0 的解析式

 

\frac{x}{cos\theta}+\frac{y}{sin\theta}=1

考虑直线x=x_0 ,不同 \theta 对应直线和这条直线都有一个交点。

 

求 x=x_0 这条直线上纵坐标的范围,也就是求 \frac{x_0}{cos\theta}+\frac{y}{sin\theta}=1 中,当 \theta 变 化时 y 的取值范围。

把直线方程化成关于 y 的函数

 

y=sin\theta(1-\frac{x_0}{cos\theta}) ,接着关于 \theta 求导

\frac{dy}{d\theta}=cos\theta(1-\frac{x_0}{cos\theta})-x_0sin^2\theta\cdot\frac{1}{cos^2\theta}

 

导函数为零时

 

cos\theta-x_0(1+tan^2\theta)=0

 

注意到 1+tan^2\theta=\frac{1}{cos^2\theta}

 

解得 cos\theta=\sqrt[3]{x_0}

 

我们知道 0\leq x_0\leq1 ,所以 0\leq\theta\leq\frac{\pi}{2}

 

因此,

 

cos^3\theta=x_0 时,经过计算 y 有最大值 y_{max}=sin^3\theta

因此领域 E 的边界是

 

(x_0,y_{max})=(cos^3\theta,sin^3\theta)

 

注意到 cos^2\theta+sin^2\theta=1

 

所以领域 E 是 x^\frac{2}{3}+y^\frac{2}{3}\leq1 在第一象限内的部分。

 

放缩回到我们所要D ,得到 (\frac{x}{8})^\frac{2}{3}+y^\frac{2}{3}\leq1 ,也就是

 

E:\frac{x^\frac{2}{3}}{4}+y^\frac{2}{3}\leq1

 

接下来积分求体积:

V=\pi\int_0^8y^2dx

 

对 E 引入参数 \alpha ,可得 x=8cos^3\alpha,y=sin^3\alpha

 

也就是 V=\pi\int_0^8y^2dx=\pi\int_0^1sin^6\alpha \;d(8cos^3\alpha)

 

=\pi\int_0^\frac{\pi}{2} -24sin^7\theta cos^2\theta \;d\theta

 

=-24\pi\int_0^\frac{\pi}{2} sin^7\theta -sin^9\theta \;d\theta

根据Wallis公式

=-24\pi\times(\frac{8!!}{9!!}-\frac{6!!}{7!!})

 

=-24\pi\times(\frac{384}{945}-\frac{48}{105})

 

=24\pi\times\frac{48}{945}=\frac{128}{105}\pi

 

本题中用到的方法叫做包络线,到了大学讲到偏导数的时候会讲更详细的公式。(所以这题如果用包络线的话要先证明一下,类似国内高考的洛必达法则那种感觉)

这里留两道思考题,一道面向大学生及以上

另一道可以作为国内高考的圆锥曲线压轴题

本题中出现的

 

x^\frac{2}{3}+y^\frac{2}{3}=1

 

叫做星形线(asteroid),有很多有趣的性质。

首先,如本题中证明的一样,它的每一条切线和 x,y 轴的两个交点的连线的距离恒定为 1

试证明,它还是内摆线的特殊情况

当小圆的半径正好为大圆半径的 \frac{1}{4} 时,小圆在大圆内滚动时,请证明圆上一个定点扫过的轨迹就是星形线。

 

(本题面向国内大学生或大学生以上的读者)

 

2. 考虑椭圆 C

 

C:\frac{x^2}{a^2}+\frac{y^2}{b^2}=1 ,其中 a+b=1,a>0,b>0

 

试求当 a,b 变化时,椭圆 C 扫过的区域。

 

(本题面向国内高中生,放到国内高考可以作为圆锥曲线压轴题。)

 参考:这里


 

东京工业大学1990年高考第二题(理科)

n 是大于等于 2 的整数

1.

请用数学归纳法证明,存在最高次数为 n-1 的多项式 P_n(x) 和最高次数为 n 的多项式Q_n(x) ,使得对于任意的 \theta ,都有以下式子成立:

\sin(2n\theta)=n\sin(2\theta)P_n(\sin^2\theta) , \cos(2n\theta)=Q_n(\sin^2\theta)

 

2.

对于 k=1,2,\cdot\cdot\cdot,n-1 ,设 \alpha_k=\frac{1}{\sin^2\frac{k\pi}{2n}}

请证明 :P_n(x)=(1-\alpha_1x)(1-\alpha_2x)\cdot\cdot\cdot(1-\alpha_{n-1}x)

3.

请证明: \sum_{k=1}^{n-1}\alpha_k=\frac{2n^2-2}{3}

我们知道在 (0,\frac{\pi}{2}) 的范围中 \sin x<x<\tan x 是恒成立的


证明方法参考三角形 OAB ,扇形 OAB ,三角形 OBC 的面积关系

 

现在我们开个脑洞,把这个强行凑到平方倒数和的形式

\sin x<x<\tan x\to\sin^2 x<x^2<\tan^2 x

也就是 \frac{1}{\tan^2 x}<\frac{1}{x^2}<\frac{1}{\sin^2 x}

注意不等式的成立范围是 (0,\frac{\pi}{2}) ,而 x 是从 1 到正无穷的整数。

这里是关键:为了让这两个区间能够相互对应,我们需要把 \frac{\pi}{2} 切割成 n 个区间,然后取第 1,2,3,\cdot\cdot\cdot,k,\cdot\cdot\cdot,n 个区间

也就是设 x=\frac{k\pi}{2n} ,其中 k=1,2,3,\cdot\cdot\cdot,n

最后再依次把这些区间内的不等式加起来

也就是

\frac{1}{\tan^2 \frac{\pi}{2n}}<\frac{4n^2}{\pi^2}<\frac{1}{\sin^2 \frac{\pi}{2n}} , \frac{1}{\tan^2 \frac{2\pi}{2n}} <\frac{4n^2}{2^2\pi^2}<\frac{1}{\sin^2 \frac{2\pi}{2n}}………………

把这些不等式全都加起来,再取一个极限就有

\lim_{n \to \infty}\sum_{k=1}^n\frac{1}{\tan^2 \frac{k\pi}{2n}}<\lim_{n \to \infty}\frac{4n^2}{\pi^2}\sum_{k=1}^n\frac{1}{k^2}<\lim_{n \to \infty}\sum_{k=1}^n\frac{1}{\sin^2 \frac{k\pi}{2n}}

变个形,再利用三角函数的公式 \frac{1}{\sin^2x}-1=\frac{1}{\tan^2x}

\lim_{n \to \infty}[(\frac{\pi^2}{4n^2}\sum_{k=1}^n\frac{1}{\sin^2 \frac{k\pi}{2n}} )-\frac{\pi}{4n}]<\lim_{n \to \infty}\sum_{k=1}^n\frac{1}{k^2}<\lim_{n \to \infty}\frac{\pi^2}{4n^2}\sum_{k=1}^n\frac{1}{\sin^2 \frac{k\pi}{2n}}

\frac{\pi}{4n}在 n 趋近无穷时极限是零

根据夹逼定理,最后转化成求 \lim_{n \to \infty}\frac{1}{n^2}\sum_{k=1}^n\frac{1}{\sin^2 \frac{k\pi}{2n}}的值

在本题我们将会证明这个极限是 \frac{2}{3}

 

 

因此 \lim_{n \to \infty}\sum_{k=1}^n\frac{1}{k^2}=\frac{2}{3}\cdot\frac{\pi^2}{4}=\frac{\pi^2}{6}

 

那么问题回来了,这个极限如何证明是 \frac{2}{3} 呢?

参考:这里


1.

证明以下等式成立

\int_0^{\frac{\pi}{2}}sin^nxdx=\int_0^{\frac{\pi}{2}}cos^n xdx

2.

令 I_n=\int_0^{\frac{\pi}{2}}sin^nxdx=\int_0^{\frac{\pi}{2}}cos^n xdx

求证 I_n=\frac{n-1}{n}I_{n-2}\quad(n\geq2)

3.

证明以下等式成立

I_n=\left\{ \begin{array}{ccc} \frac{n-1}{n}\cdot\frac{n-3}{n-2}\cdot\cdot\cdot\cdot\cdot\cdot\frac{3}{4}\cdot\frac{1}{2}\cdot\frac{\pi}{2}\\ \frac{n-1}{n}\cdot\frac{n-3}{n-2}\cdot\cdot\cdot\cdot\cdot\cdot\frac{4}{5}\cdot\frac{2}{3} \end{array} \right\} (上半段是n为偶数,下半段是n为奇数)

4.

设 S_n=\int_0^{\frac{\pi}{2}}x^2cos^{2n}xdx ,证明以下等式成立

S_{n-1}-\frac{2n}{2n-1}S_n=\frac{2}{2n(2n-1)}I_{2n}\quad(n\geq1)

5.证明以下等式成立

S_N=\frac{(2N-1)(2N-3)\cdot\cdot\cdot\cdot\cdot\cdot5\cdot3\cdot1}{2N(2N-2)\cdot\cdot\cdot\cdot\cdot\cdot4\cdot2}\cdot\frac{\pi}{4}\cdot(\frac{\pi^2}{6}-\sum_{n=1}^{N}\frac{1}{n^2})

6.证明以下等式成立

S_N\leq\frac{1}{2N+2}\cdot\frac{(2N-1)(2N-3)\cdot\cdot\cdot\cdot\cdot\cdot5\cdot3\cdot1}{2N(2N-2)\cdot\cdot\cdot\cdot\cdot\cdot4\cdot2}\;(\frac{\pi}{2})^3

7.证明以下等式成立

\sum_{n=1}^{\infty}\frac{1}{n^2}=\frac{\pi^2}{6}

 

 

这题是巴塞尔问题的初等解答,也被叫做zeta(2),是数学中很重要的一个极限。

巴塞尔问题的另一个利用三角函数的初等解答,曾经也在另一年的东京工业大学的题目中出现。

以后有机会写写

 

这题第三问证明的那个是Wallis公式(华莱士(?)公式),用来求三角函数积分的很好用的公式

1. 送分的

\int_0^{\frac{\pi}{2}}sin^nxdx=\int_0^{\frac{\pi}{2}}cos^n xdx

令 t=\frac{\pi}{2}-x

\int_0^{\frac{\pi}{2}}sin^nxdx=\int_{\frac{\pi}{2}}^0cos^nt\cdot(-1)dt=\int_0^{\frac{\pi}{2}}cos^ntdt=\int_0^{\frac{\pi}{2}}cos^nxdx

得证

2.部分积分法,看到 I_n 和 I_{n+2} 就应该想到部分积分法了

\int sin^nxdx=\int (-cosx)\prime sin^{n-1}xdx=-cosxsin^{n-1}x-\int (-cosx)\cdot cosx\cdot (n-1)sin^{n-2}xdx

=-cosxsin^{n-1}x+\int (n-1)sin^{n-2}xdx-\int (n-1)sin^{n}xdx

第一项在 (0,\frac{\pi}{2}) 积分出来是0喔

所以

nI_n=(n-1)I_{n-2}

得证

3. S_{n+1}=\frac{2n}{2n-1}S_n+\frac{n(2n-1)}{I_{2n}}

由于 I_1=\int_0^\frac{\pi}{2} sinxdx=1

I_2=\int_0^\frac{\pi}{2} sin^2xdx=\frac{\pi}{4}

套用2问的结论

原式得证

4.对 S_n 分部积分

S_n=\int_0^{\frac{\pi}{2}}x^2cos^{2n-1}x(sinx)\prime dx=0-\int_0^\frac{\pi}{2} 2xcos^{2n-1}xsinx+x^2(-sin^2x)(2n-1)cos^{2n-2}xdx

=-\int_0^\frac{\pi}{2}2xsinxcos^{2n-1}xdx-\int_0^\frac{\pi}{2}x^2(cos^2x-1)(2n-1)cos^{2n-2}xdx

注意到后半部分是S_n 和S_{n-1}

=-\int_0^\frac{\pi}{2}2xsinxcos^{2n-1}xdx-(2n-1)S_n+(2n-1)S_{n-1}

然后把 S_n 和 S_{n-1} 拿到同一侧

(2n-1)S_{n-1}-2nS_n=\int_0^\frac{\pi}{2}2xsinxcos^{2n-1}xdx

对右侧的部分进行部分积分

=\int_0^\frac{\pi}{2}\frac{2}{2n}x(-cos^{2n}x)\prime dx=-\int_0^\frac{\pi}{2}\frac{2}{2n}(-cos^{2n}x)dx=\frac{1}{n}I_{2n}

整理得到 S_{n-1}-\frac{2n}{2n-1}S_n=\frac{2}{2n(2n-1)}I_{2n}

5.利用数学归纳法证明

如果S_N=\frac{(2N-1)(2N-3)\cdot\cdot\cdot\cdot\cdot\cdot5\cdot3\cdot1}{2N(2N-2)\cdot\cdot\cdot\cdot\cdot\cdot4\cdot2}\cdot\frac{\pi}{4}\cdot(\frac{\pi^2}{6}-\sum_{n=1}^{N}\frac{1}{n^2}) 成立的话

那么由4问的结论,计 n-1=N

(2N+1)S_{N}=2(N+1)S_{N+1}+\frac{2}{2(N+1)}I_{2N+2}

 

\frac{(2N+1)(2N-1)(2N-3)\cdot\cdot\cdot\cdot\cdot\cdot5\cdot3\cdot1}{2N(2N-2)\cdot\cdot\cdot\cdot\cdot\cdot4\cdot2}\cdot\frac{\pi}{4}\cdot(\frac{\pi^2}{6}-\sum_{n=1}^{N}\frac{1}{n^2})=2(N+1)S_{N+1}+\frac{2}{2N+2}\cdot\frac{2N+1}{2N+2}\cdot\frac{2N-1}{2N}\cdot\frac{2N-3}{2N-2}\cdot\cdot\cdot\cdot\cdot\cdot\frac{3}{4}\cdot\frac{1}{2}\cdot\frac{\pi}{2} 成立

 

把 S_{N+1} 单独拿出来

2(N+1)S_{N+1}=\frac{2N-1}{2N}\cdot\frac{2N-3}{2N-2}\cdot\cdot\cdot\cdot\cdot\cdot\frac{3}{4}\cdot\frac{1}{2}\cdot\frac{\pi}{2}(\frac{2N+1}{2}(\frac{\pi^2}{6}-\sum_{n=1}^{N}\frac{1}{n^2})-\frac{2(2N+1)}{(2N+2)^2})

2(N+1)S_{N+1}=\frac{2N-1}{2N}\cdot\frac{2N-3}{2N-2}\cdot\cdot\cdot\cdot\cdot\cdot\frac{3}{4}\cdot\frac{1}{2}\cdot\frac{\pi}{2}(\frac{2N+1}{2}(\frac{\pi^2}{6}-\sum_{n=1}^{N+1}\frac{1}{n^2})

S_{N+1}=\frac{(2N+1)(2N-1)\cdot\cdot\cdot\cdot\cdot\cdot5\cdot3\cdot1}{(2N+2)2N\cdot\cdot\cdot\cdot\cdot\cdot4\cdot2}\cdot\frac{\pi}{4}\cdot(\frac{\pi^2}{6}-\sum_{n=1}^{N+1}\frac{1}{n^2})

也就是说,对 S_{n+1} 也成立

 

 

 

利用半角公式计算 S_1

\int_0^\frac{\pi}{2} x^2cos^2xdx=\int_0^\frac{\pi}{2} x^2\cdot\frac{1+cos2x}{2}dx

=\frac{\pi^3}{48}+\frac{1}{2}\int_0^\frac{\pi}{2}x^2cos2xdx

 

接下来右半部分用部分积分法

\int_0^\frac{\pi}{2}x^2cos2xdx=\frac{1}{2}\int_0^\frac{\pi}{2}x^2(sin2x)\prime dx=-\frac{1}{2}\int_0^\frac{\pi}{2}2xsin2xdx=-\frac{1}{2}\int_0^\frac{\pi}{2}x(-cos2x)\prime dx

=-\frac{\pi}{4}-\frac{1}{2}\int_0^\frac{\pi}{2}cos2xdx

注意到 cos2x 关于 x=\frac{\pi}{4} 对称,第二项积分为0

=-\frac{\pi}{4}

 

 

S_1=\frac{\pi^3}{48}-\frac{\pi}{8}

根据 S_N=\frac{(2N-1)(2N-3)\cdot\cdot\cdot\cdot\cdot\cdot5\cdot3\cdot1}{2N(2N-2)\cdot\cdot\cdot\cdot\cdot\cdot4\cdot2}\cdot\frac{\pi}{4}\cdot(\frac{\pi^2}{6}-\sum_{n=1}^{N}\frac{1}{n^2}) 计算出的 S_1 和这个数值是相等的。

 

根据数学归纳法,得证

 

6. 看到积分和小于号,就要想到被积函数的放缩啦

 

意思就是,我们要寻找一个函数,使得它积分出来的结果是

\frac{1}{2N+2}\cdot\frac{(2N-1)(2N-3)\cdot\cdot\cdot\cdot\cdot\cdot5\cdot3\cdot1}{2N(2N-2)\cdot\cdot\cdot\cdot\cdot\cdot4\cdot2}\;(\frac{\pi}{2})^3

 

并且在积分区间内恒大于 x^2cos^{2n}x

联想前几问,这个结果和wallis公式的形式很相似。

我们知道,在 (0,\frac{\pi}{2}) 的区间内,

 

 

我们把这个式子用 带有I 的算式表示出来的话

\frac{1}{2N+2}\cdot\frac{(2N-1)(2N-3)\cdot\cdot\cdot\cdot\cdot\cdot5\cdot3\cdot1}{2N(2N-2)\cdot\cdot\cdot\cdot\cdot\cdot4\cdot2}\;(\frac{\pi}{2})^3

=\frac{(2N+2)-(2N+1)}{2N+2}\cdot\frac{(2N-1)(2N-3)\cdot\cdot\cdot\cdot\cdot\cdot5\cdot3\cdot1}{2N(2N-2)\cdot\cdot\cdot\cdot\cdot\cdot4\cdot2}\;(\frac{\pi}{2})^3

=(\frac{\pi}{2})^2(I_{2N}-I_{2N+2})

那么思路就很明确了

 

 

如果我们把式子里的 x 换成 sinx 的话

那么就有

\int_0^\frac{\pi}{2} x^2cos^2nxdx\rightarrow \int_0^\frac{\pi}{2} sin^2xcos^{2n}xdx=I_{2N}-I_{2N+2}

还差两个 \frac {\pi}{2}

 

那么我们如果把 x 换成 \frac{\pi}{2}sinx 就好了啦!正好题里还给了

 

至于为什么有 x<\frac{\pi}{2}sinx 成立,证明在下面可看可不看

————————————————————————————————

我们知道, f(x)=\frac{sinx}{x} 在 (0,\frac{\pi}{2}) 的增减性可以用导数求得

f\prime(x)=\frac{cosx\cdot x-sinx}{x^2}=\frac{x-tanx}{x^2cosx}

 

分母为正数,因此我们考虑分子的正负

设 g(x)=x-tanx (x\in(0,\frac{\pi}{2}))

g\prime (x)=1-\frac{1}{cos^2x}<0 (三角函数的取值范围)

g(0)=0 ,因此 g(x)<0

因此,函数 f(x) 在 x=\frac{\pi}{2} 处取得最小值 \frac{2}{\pi}

根据 \lim_{x \to 0}\;\frac{sinx}{x}=1 ,函数 f(x) 在 (0,\frac{\pi}{2}) 的值域是

(\frac{2}{\pi},1) ,恒大于 \frac{2}{\pi}

 

因此 x<\frac{\pi}{2}sinx 在 (0,\frac{\pi}{2}) 恒成立

 

————————————————————————————————————

所以有

\int_0^\frac{\pi}{2} x^2cos^2nxdx<(\frac{\pi}{2} )^2\int_0^\frac{\pi}{2} sin^2xcos^{2n}xdx=(\frac{\pi}{2} )^2(I_{2N}-I_{2N+2})

得证

 

 

7.我们知道, 在积分区间内x^2cos^2x是大于0的,所以有S_n>0

带入5,6的结果

0<\frac{(2N-1)(2N-3)\cdot\cdot\cdot1}{2N(2N-2)\cdot\cdot\cdot2}\cdot\frac{\pi}{4}\cdot(\frac{\pi^2}{6}-\sum_{n=1}^{N}\frac{1}{n^2})\leq\frac{1}{2N+2}\cdot\frac{(2N-1)(2N-3)\cdot\cdot\cdot3\cdot1}{2N(2N-2)\cdot\cdot\cdot4\cdot2}\;(\frac{\pi}{2})^3

把一样的东西约分掉(约分掉那一长串东西是正数)

0<(\frac{\pi^2}{6}-\sum_{n=1}^{N}\frac{1}{n^2})\leq\frac{1}{N+1}\cdot(\frac{\pi}{2})^2

因为 \lim_{x\to \infty}\frac{1}{N+1}\cdot(\frac{\pi}{2})^2=0

根据夹逼定理

\lim_{0 \to \infty}\frac{\pi^2}{6}-\sum_{n=1}^{N}\frac{1}{n^2}=0

因此

\sum_{n=1}^{\infty}\frac{1}{n^2}=\frac{\pi^2}{6}

 

得证

参考:这里


 

n 是大于 1 的整数。这里把满足是某一个自然数(不包括 0 )的 n 次幂的数称之为 n 次数。

1.试证明连续两个自然数的积不是 n 次数

2.试证明连续 n 个自然数的积不是 n 次数

东京大学2012年高考第四题(理科)


 

计圆周率为 \pi ,定义以下等式

a_n=\int_0^{2-\sqrt3}\frac{1-x^{4n}}{1+x^2}\;dx

b_n=\int_0^{2-\sqrt3}\frac{1+x^{4n+2}}{1+x^2}\;dx

 

1. 请证明

\lim_{n \to \infty}a_n=\lim_{n \to \infty}b_n=\frac{\pi}{12}

 

2. 请证明 3.141<\pi<3.142

参考数值 1.7320508<\sqrt3<1.7320509

 

本题里的积分是用来计算π的数值的很重要的积分

1.

a_n=\int_0^{2-\sqrt3}\frac{1-x^{4n}}{1+x^2}\;dx=\int_0^{2-\sqrt3}\frac{1}{1+x^2}-\int_0^{2-\sqrt3}\frac{x^{4n}}{1+x^2}\;dx

对于前半部分

\int_0^{2-\sqrt3}\frac{1}{1+x^2}=[arctanx]_0^{2-\sqrt3}

 

根据常识,我们知道 15^\circ 的直角三角形的直角边比例正好是 (2-\sqrt3):1 ,因此考虑用半角公式证明这个正切值

tan30^\circ=\frac{2tan15^\circ}{1-tan^215^\circ}=\frac{1}{\sqrt3},这里设 tan15^\circ =x,等同于解二次方程 2\sqrt3x=1-x^2

x^2+2\sqrt3x-1=0,解得x=\frac{-2\sqrt3\pm\sqrt{4+12}}{2}=2\pm \sqrt3

根据 tan15^\circ <tan30^\circ , tan15^\circ=2-\sqrt3

因此

\int_0^{arctan(2-\sqrt3)}1\;d\theta=\int_0^{\frac{\pi}{12}}1 \;d\theta

=\frac{\pi}{12}

 

对于 \lim_{n \to \infty}\int_0^{2-\sqrt3}\frac{x^{4n}}{1+x^2}\;dx ,我们着眼于分母。

也就是 \lim_{n \to \infty}\int_0^{2-\sqrt3}\frac{x^{4n}}{1+x^2}\;dx<\lim_{n \to \infty}\int_0^{2-\sqrt3}\frac{x^{4n}}{x^2}\;dx

\lim_{n \to \infty}\int_0^{2-\sqrt3}\frac{x^{4n}}{x^2}\;dx=\lim_{n \to \infty}\int_0^{2-\sqrt3}x^{4n-2}\;dx

=\lim_{n \to \infty}[\frac{x^{4n-1}}{4n-1}]_0^{2-\sqrt3} =\lim_{n \to \infty}\frac{(2-\sqrt3)^{4n-1}}{4n-1}

=0

 

所以 \lim_{n \to \infty}a_n=\frac{\pi}{12}

同理可证 \lim_{n \to \infty}b_n=\frac{\pi}{12}

 

(偷偷更新了一份简洁的解答)

2.

由第一问我们可以推测出

a_1<a_2<\cdot\cdot\cdot \lim_{n \to \infty}a_n<\frac{\pi}{12}<\lim_{n \to \infty}b_n<\cdot\cdot\cdot <b_2<b_1

先计算 a_1=\int_0^{2-\sqrt3}1-x^2\;dx=2-\sqrt3-\frac{(2-\sqrt3)^3}{3}

带入 \sqrt3=1.7320509 ,后半部分因为带有三次幂,而且前面是负号所以我们带入 \sqrt3=1.732

这时 a_1>0.26153 ,把这个乘上 12 大概是 3.138,精度不够,继续计算 a_2

a_2=\int_0^{2-\sqrt3}(1+x^4)(1-x^2)\;dx

=\int_0^{2-\sqrt3}-x^6+x^4-x^2+1=-\frac{(2-\sqrt3)^7}{7}+\frac{(2-\sqrt3)^5}{5}-\frac{(2-\sqrt3)^3}{3}+(2-\sqrt3)

根据不同的次数(为了计算方便)和符号(分数前面如果是正号 \sqrt3 带上限,负号带下限。想想看,为什么)带入不同精度的 \sqrt3 ,这里我们分别带入 1.73,1.74,1.732,1.7320509

因此 a_2>0.26175 ,乘上 12 是 3.141 ,满足条件

 

对于 b_n 也一样

b_1=\int_0^{2-\sqrt3}x^4-x^2+1\;dx=\frac{(2-\sqrt3)^5}{5}-\frac{(2-\sqrt3)^3}{3}+2-\sqrt3

这里就是分数前面正号取下限,负号取上限

我们带入 1.73,1.733,1.7320508

b_1<2.619 乘 12 是 3.1428 ,精度不够继续计算 b_2

 

b_2=\int_0^{2-\sqrt3}x^8-x^6+x^4-x^2+1\;dx

=\frac{(2-\sqrt3)^9}{9}-\frac{(2-\sqrt3)^7}{7}+\frac{(2-\sqrt3)^5}{5}-\frac{(2-\sqrt3)^3}{3}+2-\sqrt3

我们带入 1.73,1.733,1.73,1.7321,1.7320508

b_2<0.26182 乘12是 3.14184 ,满足条件

因此和我们有

0.26175<a_2<\frac{\pi}{12}<b_2<0.26182

3.141<\pi<3.14184

3.141<\pi<3.142

大阪大学2013年高考附加题第二题(纯理科)


定义数列 \left\{ a_n \right\} 满足 a_n=\frac{n!}{\sqrt{n}n^ne^{-n}} ,求证 \lim_{n \to \infty}a_n=\sqrt{2\pi}

 

(1)定义数列 \left\{ b_n \right\} 满足 \frac{2^{2n}(n!)^2}{\sqrt{n}(2n)!} ,试证明 \lim_{n \to \infty}b_n=\sqrt{\pi}

注: 0<x<\frac{\pi}{2} 时 sin^{2n+1}x<sin^{2n}x<sin^{2n-1}x (n=1,2,3,\cdot\cdot\cdot) 成立这一条件可以直接使用。

 

(2)请证明对于所有自然数 n ,都有

0<\log\frac{a_n}{a_{n+1}}<\frac{100}{n(n+1)}

成立

 

(3)请证明

\lim_{n \to \infty}\frac{a_n}{a_{2n}}=1

 

(4)

请证明

\lim_{n \to \infty}a_n=\sqrt{2\pi}

本题证明的是估算阶乘的数量级的很好的一个公式(Stirling公式)

第一问

首先看到复杂的形式不要慌很重要,要好好回想和自己学过的哪些知识相关

 

看到 \frac{2^{2n}(n!)^2}{\sqrt{n}(2n)!} 这种形式,和 \sin^{2n+1}x<\sin^{2n}x<\sin^{2n-1}x

自然想到积分,也就是Wallis(华莱士)公式

 

通过部分积分法,我们可以得到递推公式

记 I_n=\int_0^\frac{\pi}{2}\sin^nxdx ,其中 n 是自然数

I_n=\int_0^{\frac{\pi}{2}}\sin^nxdx=\int_0^{\frac{\pi}{2}}(-\cos x)\prime \sin^{n-1}xdx=\int_0^{\frac{\pi}{2}}\cos x(\sin^{n-1}x)\prime dx

=(n-1)\int_0^\frac{\pi}{2}\cos^2x\sin^{n-2}xdx=(n-1)\int_0^\frac{\pi}{2}\sin^{n-2}x-\sin^nxdx

所以 I_n=(n-1)(I_{n-2}-I_n)

也就是 I_n=\frac{n-1}{n}I_{n-2}

由于我们知道只有当 n 为偶数时,积分结果才会出现 \pi

所以考虑 I_{2n} 时的情况

递推写下去,有 I_{2n}=\frac{(2n-1)(2n-3)\cdot\cdot\cdot3\cdot1}{2n(2n-2)(2n-4)\cdot\cdot\cdot4\cdot2}\cdot I_0

=\frac{(2n-1)(2n-3)\cdot\cdot\cdot3\cdot1}{2n(2n-2)(2n-4)\cdot\cdot\cdot4\cdot2}\cdot \frac{\pi}{2}

 

注意题目中要凑出 (2n)! 的形式,所以在分子分母上同时乘 2n(2n-2)(2n-4)\cdot\cdot\cdot4\cdot2 的话,就可以使分子变成我们想要的形式

也就是 I_{2n}=\frac{(2n)!}{[2n(2n-2)(2n-4)\cdot\cdot\cdot4\cdot2]^2}\cdot \frac{\pi}{2}

分母可以提出很多很多 2 ,因此

I_{2n}=\frac{(2n)!\pi}{(n!)^22^{2n+1}}

观察 b_n 的形式,我们可能需要取一个倒数

\frac{1}{I_{2n}}=\frac{(n!)^22^{2n+1}}{(2n)!\pi} ,另外 b_n=\frac{2^{2n}(n!)^2}{\sqrt{n}(2n)!}

两个式子做除法(右除左),得到

b_nI_{2n}=\frac{\pi}{2\sqrt n}

所以证明 \lim_{n \to \infty}b_n=\sqrt{\pi} ,等同于证明 \lim_{n \to \infty}\sqrt n\;I_{2n}=\frac{\sqrt\pi}{2}

式子两边为了消去根号做一个平方

也就是要证明 \lim_{n \to \infty} n\;I_{2n}^2=\frac{\pi}{4}

I_{2n}=\frac{2n-1}{2n}I_{2n-2} ,为了凑出平方,等式两边同时乘 I_{2n}

I_{2n}^2=\frac{2n-1}{2n}I_{2n-2}I_{2n}

也就是 I_{n}^2=\frac{n-1}{n}I_{n-1}I_{n}

 

注意等式右侧,接下来我们考虑数列 I_n 相邻两项的积

I_n=\frac{n-1}{n}I_{n-2} ,等式两边同时乘 I_{n-1}

I_nI_{n-1}=\frac{n-1}{n}I_{n-1}I_{n-2}

递推公式写下去

I_nI_{n-1}=\frac{(n-1)!}{n!}I_{1}I_{0}=\frac{\pi}{2n}

也就是 I_{2n}^2=\frac{2n-1}{2n}\cdot\frac{\pi}{4n}

nI_{2n}^2=\frac{2n-1}{2n}\cdot\frac{\pi}{4}

取极限,有

\lim_{n \to \infty}nI_{2n}^2=\frac{\pi}{4}

因此得证

 

第二问直接带进去就好

\log\frac{a_n}{a_{n+1}}=\log(\frac{n!}{\sqrt{n}n^ne^{-n}}\cdot\frac{\sqrt{n+1}(n+1)^{n+1}e^{-n-1}}{(n+1)!})

=\log\frac{\sqrt{n+1}(n+1)^{n+1}}{en^n\sqrt n (n+1)}

=\log\frac{1}{e}\cdot(\frac{n+1}{n})^{n+\frac{1}{2}}

=-1+(n+\frac{1}{2})[\log(n+1)-\log n]

注意到 \log(n+1)-\log n=\int_n^{n+1}\frac{1}{x} dx


考虑梯形 ADBC 和 EFBC 的面积

\frac{1}{n+\frac{1}{2}}<\frac{1}{x} dx<\frac{1}{2}(\frac{1}{n}+\frac{1}{n+1})

带入原式

得到 0<\log\frac{a_n}{a_{n+1}}<\frac{1}{4n(n+1)}

也就是 0<\log\frac{a_n}{a_{n+1}}<\frac{100}{n(n+1)}

 

第三问

考虑第二问的递推式

0<\log{a_n}-\log{a_{n+1}}<\frac{100}{n(n+1)}

0<\log{a_{n+1}}-\log{a_{n+2}}<\frac{100}{(n+1)(n+2)}

0<\log{a_{n+2}}-\log{a_{n+3}}<\frac{100}{(n+2)(n+3)}

…………………………

0<\log{a_{2n-1}}-\log{a_{2n}}<\frac{100}{(2n-1)2n}

全部加起来,可以得到

0<\log\frac{a_n}{a_{2n}}<100[\frac{1}{n(n+1)}+\frac{1}{(n+1)(n+2)}+\cdot\cdot\cdot\frac{1}{2n(2n-1)}]

对上限的分数部分裂项求和

\frac{1}{n(n+1)}+\frac{1}{(n+1)(n+2)}+\cdot\cdot\cdot\frac{1}{2n(2n-1)}

=\frac{1}{n}-\frac{1}{n+1}+\frac{1}{n+1}-\frac{1}{n+2}+\cdot\cdot\cdot+\frac{1}{2n-1}-\frac{1}{2n}

=\frac{1}{2n}

因此 0<\log\frac{a_n}{a_{2n}}<\frac{50}{n}

当 n \to \infty 时,根据夹逼定理 \lim_{n \to \infty}\log\frac{a_n}{a_{2n}}=0

也就是 \lim_{n \to \infty}\frac{a_n}{a_{2n}}=1

第四问

 

最后把 a_n 和 a_{2n} 代入

\frac{a_n}{a_{2n}}=\frac{n!}{\sqrt{n}n^ne^{-n}}\cdot\frac{\sqrt{2n}(2n)^{2n}e^{-2n}}{(2n)!}

约分,并考虑凑出 b_n 的形式

=\frac{\sqrt{2n}e^{-n}(n!)^22^{2n}n^n}{(2n)!n!\sqrt n}

=b_n\cdot\frac{\sqrt{2n}n^ne^{-n}}{n!}

观察剩下的东西

发现正好和 a_n 的形式如出一辙

所以 \frac{a_n}{a_{2n}}=\frac{\sqrt2b_n}{a_n}

也就是 \lim_{n \to \infty}\frac{a_n}{a_{2n}}=\lim_{n \to \infty}\frac{\sqrt2b_n}{a_n}

把第一问以及第三问的结果代入

可得

\lim_{n \to \infty}a_n=\sqrt{2\pi}


对于自然数 n ,考虑函数 f_n(x)=x^ne^{1-x} 和它的定积分 a_n=\int_0^1f_n(x)\;dx 。其中 e 是自然对数的底。回答以下问题。

1.

试证明在 0\leq x\leq1 的区间中, 0\leq f_n(x)\leq1 ,并且 0<a_n<1 。

2.

求 a_1 的值和当 n>1 时 a_n 和 a_{n-1} 的递推关系式。

3.

试证明对于自然数 n ,有下式成立:

\frac{a_n}{n!}=e-(1+\frac{1}{1!}+\frac{1}{2!}+\cdots+\frac{1}{n!})

4.

试证明对于任何自然数 n , n!e 都不可能是整数

大阪大学1997年高考第二题(理科)


 

计 \pi 为圆周率,考虑下面的积分

 

I_0=\pi \int_0^1sin\pi t\;dt

I_n=\frac{\pi^{n+1}}{n!} \int_0^1t^n(1-t)^nsin\pi t\;dt\;\;(n=1,2,3,\cdot\cdot\cdot)

 

1.

当n为自然数时,请利用数学归纳法证明下面不等式成立

1+\frac{x}{1!}+\frac{x^2}{2!}+\cdot\cdot\cdot+\frac{x^n}{n!}<e^x \;\;\;\;\;(x>0)

再由此式,证明下面不等式成立

I_0+uI_1+u^2I_2+\cdot\cdot\cdot+u^nI_n<\pi e^{\pi u}\;\;\;\;\;\;(u>0)

 

 

2.

请求出I_0,I_1的值,并证明下面关系式成立

I_{n+1}=\frac{4n+2}{\pi}I_n-I_{n-1}\;\;\;\;\;\;(n=1,2,\cdot\cdot\cdot)

3.

请证明 \pi 是无理数。

提示:假设 \pi=\frac{p}{q} ,再假定数列 A_n 满足 A_0=I_0,A_n=p^nI_n ,并证明这样规定的 A_0,A_1,A_2\cdot\cdot\cdot将都会是整数,最后尝试推导出矛盾即可

 

——————————————————————————————————

 

1.

第一部分

按照题目说明,用数学归纳法。

把式子放到不等号一边

设 f(x)=1+\frac{x}{1!}+\frac{x^2}{2!}+\cdot\cdot\cdot+\frac{x^n}{n!}-e^x

 

①证明 f_1(x)>0

f_1(x)=e^x-1-x\;\;,f_1(0)=0

f\prime_1(x)=e^x-1>0

因此对于任意的 x>0 ,都有 f_1(x)>0

 

②如果 f_n(x)>0 成立

那么 f\prime_{n+1}(x)=f_{n}(x)>0

我们知道 f\prime_{n+1}(0)=0

所以对于 x>0 ,都有 f_{n+1}(x)>0

 

根据数学归纳法, 对于任意的 n,f_n(x)>0恒成立

所以,1+\frac{x}{1!}+\frac{x^2}{2!}+\cdot\cdot\cdot+\frac{x^n}{n!}<e^x \;\;\;\;\;(x>0)

 

第二部分

1+\frac{x}{1!}+\frac{x^2}{2!}+\cdot\cdot\cdot+\frac{x^n}{n!}<e^x \;\;\;\;\;(x>0)

观察要证明的不等式

I_0+uI_1+u^2I_2+\cdot\cdot\cdot+u^nI_n<\pi e^{\pi u}\;\;\;\;\;\;(u>0)

考虑设 \pi u=x

那么有 1+\frac{\pi u}{1!}+\frac{(\pi u)^2}{2!}+\cdot\cdot\cdot+\frac{(\pi u)^n}{n!}<e^{\pi u} \;\;\;\;\;(u>0)

等式两边乘 \pi

1+\frac{\pi^2 u}{1!}+\frac{\pi^3 u^2}{2!}+\cdot\cdot\cdot+\frac{\pi^{n+1} u^n}{n!}<\pi e^{\pi u} \;\;\;\;\;(u>0)

那么如果我们能证明 \frac{\pi^{n+1}}{n!}>I_n 成立,那么就可以证出来了

要证明 \frac{\pi^{n+1}}{n!}>\frac{\pi^{n+1}}{n!} \int_0^1t^n(1-t)^nsin\pi t\;dt\;\;(n=1,2,3,\cdot\cdot\cdot)

也就是证明 \int_0^1t^n(1-t)^nsin\pi t\;dt\;\;(n=1,2,3,\cdot\cdot\cdot)<1

我们知道, t^n(1-t)^nsin\pi t<1 在 (0,1) 恒成立( x\neq \frac{1}{2} 时,乘积的每一部分都小于等于1,且 x=1 时也小于1)

所以 \int_0^1t^n(1-t)^nsin\pi t\;dt\;\;(n=1,2,3,\cdot\cdot\cdot)<\int_0^1t\;dt\;\;(n=1,2,3,\cdot\cdot\cdot)<1

因此 I_n< \frac{\pi^{n+1}}{n!}

所以

I_0+uI_1+u^2I_2+\cdot\cdot\cdot+u^nI_n<1+\frac{\pi^2 u}{1!}+\frac{\pi^3 u^2}{2!}+\cdot\cdot\cdot+\frac{\pi^{n+1} u^n}{n!}<\pi e^{\pi u}\;\;\;\;\;\;(u>0)

得证

2.

I_0=\pi \int_0^1sin\pi t\;dt=[-cos\pi t ]^1_0=2

I_1=\pi^2 \int_0^1t(1-t)sin\pi t\;dt\;\;

=\pi^2 \int_0^1tsin\pi t\;dt-\pi^2 \int_0^1t^2sin\pi t\;dt\;\;

=\pi \int_0^1t(-cos\pi t)\prime \;dt\;\;-\pi \int_0^1t^2(-cos\pi t)\prime \;dt (分部积分法)

=\pi [-tcos\pi t]^1_0-\pi \int_0^1-cos\pi t\;dt-\pi [-t^2cos\pi t]^1_0-\pi \int_0^12t(-cos\pi t)\;dt

=\pi+[sin\pi t]_0^1-\pi+2\int_0^1t(sin\pi t)\prime \;dt (分部积分法)

=2[sin\pi t\cdot t]_0^1-\int_0^12sin\pi t \;dt

=\frac{2}{\pi}[cos\pi t]_0^1=\frac{4}{\pi}

 

同样用分部积分法

I_{n+1}=\frac{\pi^{n+2}}{(n+1)!} \int_0^1t^{n+1}(1-t)^{n+1}sin\pi t\;dt

=\frac{\pi^{n+1}}{(n+1)!} \int_0^1t^{n+1}(1-t)^{n+1}(-cos\pi t)\prime \;dt

=0-\frac{\pi^{n+1}}{(n+1)!} \int_0^1[(n+1)t^{n}(1-t)^{n+1}-(n+1)t^{n+1}(1-t)^n](-cos\pi t) \;dt ,整理合并同类项

=\frac{\pi^{n}}{n!} \int_0^1t^{n}(1-t)^n(1-2t)(sin\pi t)\prime \;dt

=0-\frac{\pi^{n}}{n!} \int_0^1[nt^{n-1}(1-t)^n(1-2t)-nt^n(1-t)^{n-1}(1-2t)-2t^n(1-t)^n]sin\pi t\;dt ,整理合并同类项

=-\frac{\pi^{n}}{n!} \int_0^1t^{n-1}(1-t)^{n-1}[n(1-t)(1-2t)-nt(1-2t)-2t(1-t)]sin\pi t\;dt

=-\frac{\pi^{n}}{n!} \int_0^1t^{n-1}(1-t)^{n-1}[n(1-3t+2t^2-t+2t^2)-2t+2t^2]sin\pi t\;dt

=-\frac{\pi^{n}}{(n-1)!} \int_0^1t^{n-1}(1-t)^{n-1}[(4t^2-4t+1)]sin\pi t\;dt+\frac{2I_n}{\pi}

=-\frac{4\pi^{n}}{(n-1)!} \int_0^1t^{n-1}(1-t)^{n-1}[(t^2-t)]sin\pi t\;dt-I_{n-1}+\frac{2I_n}{\pi}

=\frac{4\pi^{n}}{(n-1)!} \int_0^1t^{n}(1-t)^{n}sin\pi t\;dt-I_{n-1}+\frac{2I_n}{\pi}

=\frac{4n}{\pi}I_{n}-I_{n-1}+\frac{2I_n}{\pi}

=\frac{4n+2}{\pi}I_n-I_{n-1}

得证

 

 

3.

按照提示我们假设 \pi=\frac{p}{q} 是有理数

那么 I_{n+1}=q\frac{4n+2}{p}I_n-I_{n-1}

定义A_n=p^{n}I_{n}

那么 A_{n+1}=qp^{n}(4n+2)I_n-p^{n+1}I_{n-1}=q(4n+2)A_n-p^2A_{n-1}

因为 A_0=2, A_1=4q 都为正整数,且 p,q,4n+2 也均为正整数

所以所有的 A_n 都是正整数

根据第一题的结论

I_0+uI_1+u^2I_2+\cdot\cdot\cdot+u^nI_n<\pi e^{\pi u}\;\;\;\;\;\;(u>0)

设 u=p ,那么有

A_0+A_1+\cdot\cdot\cdot+A_n<\frac{p}{q}e^{\frac{p}{q} p}

当 n \to \infty 时,不等号左侧是正整数的和也 \to \infty

但不等号右侧是一个常数,不含 n

因此导出矛盾,之前的假设不成立

 

所以 ,\pi 是无理数

大阪大学2003年高考第四题(理科)


 

记质数有小到大排序得到的无穷数列为

p_1,p_2,\cdots,p_n,\cdots

1.

试证明对于自然数 n ,下式成立

\sum_{k=1}^n\frac{1}{k}<\frac{1-(\frac{1}{p_1})^{n+1 }}{1-\frac{1}{p_1}}\times\frac{1-(\frac{1}{p_2})^{n+1 }}{1-\frac{1}{p_2}}\times\cdots\frac{1-(\frac{1}{p_n} )^{n+1 }}{1-\frac{1}{p_n} }

2.

试证明这个无限数列发散

\sum_{k=1}^\infty[-\log(1-\frac{1}{p_k} )]

3.

试证明 \sum_{k=1}^\infty\frac{1}{p_k} 发散

大阪大学2016年高考附加题第二题(纯理科)


考虑定义域为全体实数,且不一定连续的函数 f(x) 。假设 f(x) 满足:

f(x+y)=f(x)+f(y)
f(xy)=f(x)f(y)
f(1)=1


试证明:

对于任何有理数 x ,都有 f(x)=x
对于实数 x,y ,如果 x\leq y 则 f(x)\leq f(y)
对于任何实数 x ,都有 f(x)=x

大阪大学2015年高考附加题第一题(纯理科)


1.

已知 f(t) 在 0\leq t\leq1 上连续,当 \tan x =t 时,试证明:

\int_0^\frac{\pi}{4}\frac{f(\tan x)}{\cos^2x}dx=\int_0^1f(t)\;dt

2.

已知 n 是非负整数。 请利用上一问的结论,求出 \int_0^\frac{\pi}{4}\frac{\tan^nx}{\cos^2x}dx 的值

另外,请证明 \int_0^\frac{\pi}{4}\tan^nx\;dx\leq\frac{1}{n+1}

3.

对于非负整数 n 和满足 0\leq x\leq\frac{\pi}{4} 的 x ,试证明:

\frac{1-\tan^2x+\tan^4x-\cdots+(-1)^n\tan^{2n}x}{\cos^2x}=1-(-1)^{n+1}\tan^{2(n+1)}x

4.

请利用第二问和第三问的结论,求出 \lim_{n \to \infty}\sum_{k=0}^n(-1)^k\frac{1}{2k+1} 的值

 

本身题目还是很简单的啦……只是最后一问的 \frac{π}{4}=1-\frac{1}{3}+\frac{1}{5}-\frac{1}{7}+\frac{1}{9} ……

这个结论还是挺有意思的,虽然收敛很慢没法用来求圆周率的值……

金泽大学2012年高考第三题(理科)


对于数列的求和公式

\sum_{k=1}^nk=\frac{1}{2}n(n+1) , \sum_{k=1}^nk^2=\frac{1}{6}n(n+1)(2n+1) , \sum_{k=1}^nk^3=[\frac{1}{2}n(n+1)]^2

这里,我们考虑一般情况下的结论。 p,n 是自然数。

 

(1)

试证明:存在一个最高次数为 p+1 的多项式 S_p(x) ,它满足 \sum_{k=1}^nk^p=S_p(n) 。

 

(2)

设 q 是自然数,考虑在(1)中定义的多项式 S_1(x),S_3(x),S_5(x),\cdot\cdot\cdot S_{2q-1}(x) 。

如果

\sum_{j=1}^qa_jS_{2j-1}(x)=x^q(x+1)^q

是恒等式,请用 q 表示出 a_1,a_2,\cdot\cdot\cdot,a_q

 

(3)

设 q 是大于等于 2 的自然数。考虑在(1)中定义的多项式 S_2(x),S_4(x),S_6(x),\cdot\cdot\cdot S_{2q-2}(x) 。

如果

\sum_{j=1}^{q-1}b_jS_{2j}(x)=x^{q-1}(x+1)^{q-1}(cx+q)

是恒等式,请用 q 表示出 c 和 b_1,b_2,\cdot\cdot\cdot,b_{q-1} 。

 

(4)

设 p 是大于等于 3 的奇数。请证明

\frac{dS_p(x)}{dx}=pS_{p-1}(x)

 

 

 

——————————————————————————————————————

 

之前所有文章的答案已经补完(包括物理和化学)

另外,笔者在下面这篇文章里加了两道思考题

一道国内高考圆锥曲线压轴难度,一道大学生应该能解的难度。

 

一根木棒从墙角滑落时扫过的区域是??(保持两端分别在直角两边上)——大阪大学2011年高考第二题(理科)

 

注意本题最后一问

 

(1^3+2^3+3^3+……+x^3)\prime=3(1^2+2^2+3^2+……+x^2)

(1^5+2^5+3^5+……+x^5)\prime=5(1^4+2^4+3^4+……+x^4)

 

相邻次数的求和公式竟然通过导数就这么连接了起来

很神奇吧

 

 

————————————————————————————————————————

 

第一问思路很明显也很简单 就是数学归纳法

 

假设 p=1,2,3,\cdot\cdot\cdot,m-1 时都有满足条件的多项式 S_p(x)

 

考虑 (s+1)^{m+1}-s^{m+1} ,根据二项定理展开

=(m+1)s^m+\sum_{t=1}^{m-1}C^{m+1}_ts^{t}

代入 s=1,2,\cdot\cdot\cdot,k 并求和(2^{m+1}-1^{m+1})+(3^{m+1}-2^{m+1})+\cdot\cdot\cdot+[(k+1)^{m+1}-k^{m+1}] =(m+1)\sum_{s=1}^k s^m+\sum_{s=1}^k\sum_{t=1}^{m-1}C^{m+1}_ts^{t}

 

对于\sum_{s=1}^k\sum_{t=1}^{m-1}C^{m+1}_ts^{t} ,交换求和顺序

=\sum_{t=1}^{m-1}C^{m+1}_tS_t(k)

 

也就是 \sum_{s=1}^k s^m=\frac{1}{m+1}[(k+1)^{m+1}-\sum_{t=1}^{m-1}C^{m+1}_tS_t(k)-1]

等式右边最高次数为 m+1

另外,由于 \sum_{k=1}^nk=\frac{1}{2}n(n+1) 存在,根据数学归纳法

原题得证

第二问

 

 

由于出现了和的关系,考虑取 x-1 之后做差

\sum_{j=1}^qa_jS_{2j-1}(x)=x^q(x+1)^q

\sum_{j=1}^qa_jS_{2j-1}(x-1)=x^q(x-1)^q

上面减下面得到

\sum_{j=1}^qa_jx^{2j-1}=x^q[(x+1)^q-(x-1)^q]

根据二项定理,显然

x^q[(x+1)^q-(x-1)^q]=2C^q_{q-1}x^{2q-1}+2C^q_{q-3}x^{2q-3}+\cdot\cdot\cdot

另外 \sum_{j=1}^qa_jx^{2j-1}=a_qx^{2q-1}+a_{q-1}x^{2q-3}+\cdot\cdot\cdot

比对多项式系数可得

a_{q-i}=2C^q_{q-2i-1}

设 q-i=j

那么有 a_j=C^q_{2j-q-1}

 

注意到 2j-q-1\geq0

j\geq\frac{q+1}{2}

显然当 j<\frac{q+1}{2} 时, a_j=0

 

所以答案为

当 j<[\frac{q+2}{2}] 时, a_j=0

当 q\geq j\geq[\frac{q+2}{2}] 时, a_j=C^q_{2j-q-1}

第三问

 

和第二问一样,利用

\sum_{j=1}^qb_jS_{2j}(x)=x^{q-1}(x+1)^{q-1}(cx+q)

\sum_{j=1}^qb_jS_{2j}(x-1)=(x-1)^{q-1}x^{q-1}(cx-c+q)

 

第一个式子减第二个式子

\sum_{j=1}^qb_jx^{2j}=x^{q-1}[(x+1)^{q-1}-(x-1)^{q-1}](cx+q)+c(x-1)^{q-1}x^{q-1}

=cx^{q-1}[x(x+1)^{q-1}-x(x-1)^{q-1}+(x-1)^{q-1}]+qx^{q-1}[(x+1)^{q-1}-(x-1)^{q-1}]

=c[x^q(x+1)^{q-1}-x^{q-1}(x-1)^{q}]+qx^{q-1}[(x+1)^{q-1}-(x-1)^{q-1}]

 

对于 x^q(x+1)^{q-1}-x^{q-1}(x-1)^{q} ,同样二项定理展开

 

x^q(x+1)^{q-1}-x^{q-1}(x-1)^{q} =x^q(x^{q-1}+C^{q-1}_{q-2}x^{q-2}+C^{q-1}_{q-3}x^{q-3}+\cdot\cdot\cdot)-x^{q-1}(x^q-C^{q}_{q-1}x^{q-1}+C^{q}_{q-2}x^{q-2}\cdot\cdot\cdot)

=(x^{2q-1}+C^{q-1}_{q-2}x^{2q-2}+C^{q-1}_{q-3}x^{2q-3}+\cdot\cdot\cdot)-(x^{2q-1}-C^{q}_{q-1}x^{2q-2}+C^{q}_{q-2}x^{2q-3}\cdot\cdot\cdot)

=[(C^{q-1}_{q-2}+C^q_{q-1})x^{2q-2}+(C^{q-1}_{q-4}+C^q_{q-3})x^{2q-4}+\cdot\cdot\cdot]+ [(C^{q-1}_{q-3}-C^q_{q-2})x^{2q-3}+(C^{q-1}_{q-5}-C^q_{q-2})x^{2q-5}+\cdot\cdot\cdot]

接下来消去奇数项。我们知道组合数 C 的公式:

C^r_n=C^{r-1}_n+C^{r-1}_{n-1} ,因此 -C^{r-1}_n=C^{r-1}_{n-1}-C^r_n

把这个式子利用到奇数项的系数,所以

x^q(x+1)^{q-1}-x^{q-1}(x-1)^{q}=[(C^{q-1}_{q-2}+C^q_{q-1})x^{2q-2}+(C^{q-1}_{q-4}+C^q_{q-3})x^{2q-4}+\cdot\cdot\cdot]+ [(-C^{q-1}_{q-2})x^{2q-3}+(-C^{q-1}_{q-4})x^{2q-5}+\cdot\cdot\cdot]

到这里思路就很明显了

在第一问中我们已经计算过

(x+1)^{q-1}-(x-1)^{q-1}=2C^{q-1}_{q-2}x^{q-2}+2C^{q-1}_{q-4}x^{q-4}+\cdot\cdot\cdot

把这两个式子代入,可以知道

c[x^q(x+1)^{q-1}-x^{q-1}(x-1)^{q}]+qx^{q-1}[(x+1)^{q-1}-(x-1)^{q-1}]

=c[(C^{q-1}_{q-2}+C^q_{q-1})x^{2q-2}+(C^{q-1}_{q-4}+C^q_{q-3})x^{2q-4}+\cdot\cdot\cdot]+ c[(-C^{q-1}_{q-2})x^{2q-3}+(-C^{q-1}_{q-4})x^{2q-5}+\cdot\cdot\cdot]+q(2C^{q-1}_{q-2}x^{2q-3}+2C^{q-1}_{q-4}x^{2q-5}+\cdot\cdot\cdot)

所以

\sum_{j=1}^qb_jx^{2j}=(2q-c)(2C^{q-1}_{q-2}x^{2q-3}+2C^{q-1}_{q-4}x^{2q-5}+\cdot\cdot\cdot)+ c[(C^{q-1}_{q-2}+C^q_{q-1})x^{2q-2}+(C^{q-1}_{q-4}+C^q_{q-3})x^{2q-4}+\cdot\cdot\cdot]

 

注意,奇数次项的系数为 0 ,且组合数不为 0

所以 c=2q 必须成立

 

接着计算 b_j ,显然有

2j=2q-i

b_j=c(C^{q-1}_{q-i}+C^{q}_{q+1-i})

带入 i ,可知

b_j=c(C^{q-1}_{2j-q}+C^{q}_{2j-q+1})

由于 c=2q ,可得

b_j=2q(C^{q-1}_{2j-q}+C^{q}_{2j-q+1})

下面考虑 C^{q-1}_{2j-q}+C^{q}_{2j-q+1} 的值,计算可得

C^{q-1}_{2j-q}+C^{q}_{2j-q+1}=\frac{(q-1)!}{(2j-q)!(2q-2j-1)!}+\frac{q!}{(2j-q+1)!(2q-2j-1)!}

通分

=\frac{(q-1)!(2j-q+1)}{(2j-q+1)!(2q-2j-1)!}+\frac{q!}{(2j-q+1)!(2q-2j-1)!}

合并同类项

=\frac{(q-1)!(2j+1)}{(2j-q+1)!(2q-2j-1)!}

带入 b_j ,并化回组合数的形式

b_j=\frac{q!(2j+1)}{(2j-q+1)!(2q-2j-1)!}

也就是 b_j=(2j+1)C^q_{2j-q+1}

同第二问,接下来求 b_j=0 的范围

由 2j-q+1\geq0 ,可以解得 j\geq\frac{q-1}{2}

所以当 j\leq\frac{q-1}{2} ,也就是 j<[\frac{q}{2}] 时, b_j=0 ,其中 [\;\;\;] 是高斯符号

综上所述,第三问答案为

c=2q ,且

当 j<[\frac{q}{2}] 时, b_j=0

当 [\frac{q}{2}]\leq j\leq q-1 时, b_j=(2j+1)C^q_{2j-q+1}

 

 

最后来到第四问,之前的第二问和第三问都是为了这一问所铺垫的

为了得到 \frac{dS_p(x)}{dx} 形式的式子

根据第二问的 \sum_{j=[\frac{q+2}{2}]}^qa_jS_{2j-1}(x)=x^q(x+1)^q

对等式两边求导

\sum_{j=[\frac{q+2}{2}]}^q\frac{d[a_jS_{2j-1}(x)]}{dx}=qx^{q-1}(x+1)^q+qx^q(x+1)^{q-1}

也就是\sum_{j=[\frac{q}{2}]}^q\frac{d[a_jS_{2j-1}(x)]}{dx}=qx^{q-1}(x+1)^{q-1}(2x+1)

 

根据第三问, \sum_{j=[\frac{q}{2}]}^{q-1}b_jS_{2j}(x)=qx^{q-1}(x+1)^{q-1}(2x+1)

我们惊奇的发现上面两个等式是一模一样

所以 \sum_{j=[\frac{q+2}{2}]}^q\frac{d[a_jS_{2j-1}(x)]}{dx}=\sum_{j=[\frac{q}{2}]}^{q-1}b_jS_{2j}(x) ,其中 q\geq2

 

注意由第三问,当 j\geq[\frac{q}{2}] 时, b_j=(2j+1)C^q_{2j-q+1}

另一方面, a_j=C^q_{2j-q-1} ,因此 a_{j+1}=C^q_{2j-q+1}

所以可得 b_j=(2j+1)a_{j+1} ,代入求导后的式子

\sum_{j=[\frac{q+2}{2}]}^q\frac{d[a_jS_{2j-1}(x)]}{dx}=\sum_{j=[\frac{q}{2}]}^{q-1}(2j+1)a_{j+1}S_{2j}(x)

注意求和符号的性质变化等式右侧,也就是 \sum_{j=[\frac{q+2}{2}]}^q\frac{d[a_jS_{2j-1}(x)]}{dx}=\sum_{j=[\frac{q+2}{2}]}^{q}(2j-1)a_{j}S_{2j-2}(x)

合并同类项可知

\sum_{j=[\frac{q+2}{2}]}^qa_j\left\{ \frac{d[S_{2j-1}(x)]}{dx}-(2j-1)S_{2j-2}(x)\right\}=0 ,其中 q\geq2

注意上式是求和递推式,从次数低的情况一层一层考虑。

 

我们知道 S_3(x)=\frac{x^4+2x^3+x^2}{4} ,对其求导并合并同类项

\frac{d[S_3(x)]}{dx}=\frac{x(x+1)(2x+1)}{2}=3S_2(x)

所以当 p=3 时成立

 

 

设 q=3 ,那么有 \sum_{j=2}^3a_j\left\{ \frac{d[S_{2j-1}(x)]}{dx}-(2j-1)S_{2j-2}(x)\right\}=0

我们知道 \frac{d[S_3(x)]}{dx}=3S_2(x) 成立,所以有

a_3\left\{ \frac{d[S_{5}(x)]}{dx}-5S_{4}(x)\right\} ,因此 \frac{d[S_5(x)]}{dx}=5S_4(x)

 

设 q=5 ,同理可证 \frac{d[S_7(x)]}{dx}=7S_6(x)

 

………………

 

同理可证, \frac{d[S_9(x)]}{dx}=9S_8(x),\frac{d[S_{11}(x)]}{dx}=11S_{10}(x)

 

所以, \frac{dS_p(x)}{dx}=pS_{p-1}(x) 恒成立

东京大学2006年高考第三题(理科)


tan1^\circ 是有理数吗?

 

思路:

我们知道,如果两个数都是有理数,那么它们的商也一定是有理数

那么关键来了,还记得正切的和角公式么

没错!和角公式里没有出现根号之类的东西

 

证:

假设 tan1^\circ 是有理数

那么根据正切的和角公式我们可以得到

tan2^\circ=tan(1^\circ+1^\circ)=\frac{tan1^\circ+tan1^\circ}{1-(tan1^\circ)^2}

因为分子分母都是有理数,那么 tan2^\circ 也是有理数

同理,

tan3^\circ=tan(1^\circ+2^\circ)=\frac{tan1^\circ+tan2^\circ}{1-tan1^\circ \times tan2^\circ}

也就是说, tan3^\circ 也是有理数

………………

………………

………………

那么, tan30^\circ 也是有理数

然而, tan30^\circ=\frac{1}{\sqrt3} 是无理数

 

因此,先前的假设不成立

tan1^\circ 为无理数

证毕

京都大学2006年高考第六题(理科)


 

1.对于 m\geq0 , n\geq1 的整数 m,n ,定义

a_{m,n}=\int_0^\pi\theta^m\cos n\theta\;d\theta
b_{m,n}=\int_0^\pi\theta^m\sin n\theta\;d\theta
试证明

a_{m+1,n}=-\frac{m+1}{n}b_{m,n}
b_{m+1,n}=(-1)^{n+1}\frac{\pi^{m+1}}{n}+\frac{m+1}{n}a_{m,n}
2.把长度为 \pi 的绳子的一端固定在半径为 1 的球面上一点在球的外面移动时,绳子扫过的区域的体积是多少?

第一题部分积分

\int_0^\pi\theta^m\cos n\theta\;d\theta

东京工业大学2000年高考第二题


 

伽罗瓦留下的遗物:日本史上最难的高考题!!——东京大学1998年高考第三题(理科)

题目背景:

日本有很多为了考大学而开设的补课班。补课班A拿到题目之后不久,便放弃了当天做出答案的想法并下班。第二天,日本国内所有的补课班(包括河合塾、駿台等顶级补课班)仍然解不出来,这时补课班B给补课班A电话,催促补课班A快点把这道题做出来。补课班A拜托安田教授解题,但仍然解不出来。最后安田教授联系了法国某数学系大学教授,专攻数学奥赛的C教授,又花了两天才把答案写出来。

当年的东京大学考生,最后只有两人做出这道题(情报不确定)。把群论知识运用到初等题目上,这是第一例。

————————————————————————————————————

 

 计 G=(V,W) 是由“具有有限个点的集合 V= \{ P_1,P_2,\cdot\cdot\cdot P_n \} ”和“在这些点之间连接的线段 W=\{E_1,E_2,\cdot\cdot\cdot,E_m\} ” 组成的图形。每一条边 E_j 都有两个顶点 P_{i_1},P_{i_2}(i_1\neq i_2) 。不考虑除了顶点之外的线段的交点。另外,每一个点都染有白色或黑色。

 例如下面的图 1 中, n=5,m=4 。边 E_i\;(i=1,2,\cdot\cdot\cdot ,4) 的顶点是 P_i 和 P_5 。另外, P_1,P_2 是白色顶点, P_3,P_4,P_5 是黑色顶点。

 作为起点的图形 G_1 (图 2)中, n=1,\;m=0 。其中唯一的顶点是白色的。

 定义以下两种操作,使原来的图形 G=(V,W) 变换为新的图形 G\prime=(V\prime,W\prime) 。每种操作都会让图形的边数和顶点数各增加 1 。

操作1:
 首先选定 G 的一个顶点 P_{i_0} 。设 V\prime 是由 V 增加一个顶点 P_{n+1} 而得到的集合。 W\prime 是由 W 增加一条边 E_{n+1} 得到的集合。在操作中, E_{n+1} 的两个顶点分别是 P_{i_0} 和 P_{n+1} 。 G\prime 其余的边的顶点都和 G 完全相同。如果在 G 中的 P_{i_0} 的颜色是白色(或黑色)的话,在 G\prime 中这个点的颜色就变为黑色(或白色)。除此以外的点的颜色都不变。另外, P_{n+1} 为白色顶点。(示例:图3)

操作2:
 首先选定 G 的一条边 E_{j_0} ,设 V\prime 是由 V 增加一个顶点 P_{m+1} 而得到的集合。 W\prime 是由 W 删除一条边 E_{j_0} ,并增加两条边 E_{i_1},\;E_{i_2} 而得到的集合。如果设 E_{j_0} 的两个顶点分别是 P_{i_1},P_{i_2} ,那么对于增加的两条边, E_{i_1} 的两个顶点分别为 P_{i_1} 和 P_{m+1} ,而 E_{i_2} 的两个顶点分别为 P_{i_2} 和 P_{m+1} 。 G\prime 其余的边的顶点都和 G 完全相同。如果在 G 中的顶点 P_{i_1} 是白色(或黑色)的话,在 G\prime 中这个点的颜色就变为黑色(或白色)。 对于P_{i_2} ,也做一样的颜色变换。除此以外的点的颜色都不变。另外, P_{n+1} 为白色顶点。

(示例:图4)

 由起点图形 G_1 出发,通过这两种操作(有限次)的变换能够得到的图形叫做“可能图形”。请回答以下问题。

 

试证明图 5 的三个图形,都是“可能图形”。(注意,这三个图形所有的顶点都是白色。)
设 n 为自然数,求有 n 个顶点的棒状图形作为“可能图形”的充分必要条件。(如图6。注意,所有的顶点都是白色)

 

 

 

考试时间:三道题150分钟,也就是说这道题50分钟

————————————————————————————————————

宣传一波物理专栏:来看看日本高中的高考理科都考些啥?顺便求关注。

————————————————————————————————————

 

 

评论区果然厉害,
@灵剑

@Recursion
都有很厉害的解法。既然这样的话笔者就不放答案里的麻烦解法了…………

 

第一问直接构造就行,主要考察是否理解了问题吧……第一个图是从起始点连续执行1两次,第二个是从起始点连续执行1四次,第三个先执行1,再执行2得到白白黑,然后对中间的白执行两次1,对右边的黑执行三次1
第二问关键是观察两种操作下图形的某种不变性。我们可以看到,要消除一条线上的黑色顶点,只有两种方法:一种是移到边上,然后执行操作1;一种是把两个黑色顶点凑到一起,然后执行操作2,。这提示我们用黑色顶点之间的距离来定义不变量。对于任意一种分布,我们在两边各补上一个黑色顶点,然后计算:从第一个顶点(最左边补上的)到第二个顶点的距离(相邻两个顶点距离定义为1,隔一个为2,依此类推),加上第三个到第四个的距离,加上第五个到第六个的距离……记为距离A;从第二个到第三个的距离,加上第四个到第五个的距离,加上第六个到第七个的距离……记为距离B。不难发现距离A + 距离B = n + 1。全部顶点都是白色的时候,距离A = n + 1,距离B = 0。下面的讨论中我们并不仔细区分这两种距离,只要注意它们永远是互补的就行,任意两个黑色顶点的距离(包括到两端补上的顶点的)总体现在其中一种距离中。
显然我们只能对线的两端使用操作1,则:
1. 对线的最右端使用操作1的时候,如果最右边为黑色顶点,则其中一种距离增加2,另一种减少1;如果最右边为白色顶点,同样其中一种距离增加2,另一种减少1。
2. 对线的最左端使用操作1的时候,同上,其中一种距离增加2,另一种减少1,然后两个距离互换。在不考虑两个距离的顺序的时候,可以认为仍然是一种距离增加2,另一种减少1
3. 对某条边使用操作2的时候,分类讨论不难发现,仍然是某种距离增加2,而另一种减少1

注意到无论是增加2,还是减少1,对3取余数的结果都是减少1(从2变为1,从1变为0,从0则变为2),因此两种距离无论使用任何操作,对3取余数的结果都是每次减1,并且在同余的范围内循环。
前面说过,全部顶点都是白色的时候,两种距离中有一种为0,因此对3的余数也必须为0。注意到初始状态下有一个顶点,一种距离为2,另一种距离为0,可以得到,当有n个顶点时,一种距离在模3的意义下与3-n同余,另一种距离与1-n同余,因此3-n和1-n中至少有一个为3的倍数,所以n必须是3的倍数,或者n模3余1,即n必须为3k或3k+1的形式。
上面证明了必要性,下面通过构造法证明充分性。首先n=1是可行的。对于初始节点连续进行两次操作1得到3个连续白色顶点;先进行操作1,然后进行一次操作2,再对唯一的黑色顶点进行操作1,得到4个连续白色顶点,证明了n=3,n=4可行。如果n=m可行,则:对于最右顶点执行1;对于最右的边执行2;对于最右顶点(黑色)执行1。则得到了n=m+3的可行解。用数学归纳法可以证明,n=3k, n=3k+1都可行。

by 灵剑

 

 

我们定义一个变换X为新建一个n-1个点的图,若第i条边连接的两个点同色,则变换后点i为白色,否则为黑色。
定义X的逆变换为X^{-1}。
在X^{-1}变换后的情况下考虑操作1和2。
操作1就是将末尾的点反色后再添加一个同色点。
考虑操作2,设原来颜色在X^{-1}意义下是abc,操作后就是aBBc(b与B反色)。
那么容易发现每次操作后黑白两种颜色数量差在模3意义下不变。
初始时黑白色差显然不为0,所以n+1=3k,也就是n=3k-1是不合法的。
n=3k和3k-2归纳法构造一下就行了吧。。。
by Recursion

 

 

至于日本人的解法证明3n+2不可能……他们的思路是这样的:

1.证明操作1和操作2的顺序就算交换,也会得到一样的结果

2.证明操作1之后,一定会变成OXX……XOXX……XO,或者OXX……X,或者XX……XO的形式。其中O代表白球,X代表黑球,X的个数可能为零。

3.设O=120°的旋转矩阵A,X=关于x轴对称的变换矩阵B。

注意:操作2不会让矩阵的乘积发生变化

所以进行完所有的操作1后,矩阵的乘积一定要是A^2才行(注意到A^3=E)

然而进行完所有的操作1后,矩阵的乘积不可能是A^2。

 

感觉没有评论区的证法简单,这里就不赘述了

posted on 2018-07-18 23:05  Eufisky  阅读(1711)  评论(0编辑  收藏  举报

导航